Site Loader

Содержание

Постоянный электрический ток: определение, механизм, характеристики

Определение 1

Постоянный ток – это упорядоченное движение заряженных частиц, движущихся в одном направлении.

По теории данные заряженные частицы относят к носителям тока. В проводниках и полупроводниках такими носителями являются электроны, в электролитах – заряженные ионы, в газах – электроны и ионы. Металлы характеризуются перемещением только электронов. Отсюда следует, что электрический ток в них – это движение электронов проводимости.

Результат прохождения электрического тока в металлах и электропроводящих растворах заметно отличается. Наличие химических процессов в металлах при протекании тока отсутствует. В электролитах под воздействием тока происходит выделение ионов вещества на электродах. Различие заключается в отличии носителей зарядов металла и электролита. В металлах – это свободные электроны, отделившиеся от атомов, в растворах – ионы, атомы или их группы с зарядами.

Необходимые условия существования электрического тока

Первое необходимое условие существования электрического тока любого вещества – наличие носителей заряда.

Для равновесного состояния зарядов необходимо равнение нулю разности потенциалов между любыми точками проводника. При нарушении данного условия, заряд не сможет переместиться. Отсюда следует, что второе необходимое условие существования электрического тока в проводнике – создание напряжения между некоторыми точками.

Определение 2

Упорядоченное движение свободных зарядов, возникающее в проводнике как результат воздействия электрического поля, называют током проводимости.

Такое движение возможно при перемещении в пространстве заряженного проводника или диэлектрика. Подобный электрический ток получил название конвекционного.

Механизм осуществления постоянного тока

Для постоянного прохождения тока в проводнике следует подсоединить к проводнику или их совокупности устройство, в котором постоянно происходит процесс разделения электрических зарядов для поддержания напряжения в цепи. Данный механизм получил название источника тока (генератора).

Силы, разделяющие заряды, называют сторонними. Они характеризуются неэлектрическим происхождением, действуют внутри источника. При разделении зарядов сторонние силы способны создать разность потенциалов между концами цепи.

Если электрический заряд перемещается по замкнутой цепи, то работа электростатических сил равняется нулю. Отсюда следует, что суммарная работа сил A, действующих на заряд, равна работе сторонних Ast. Определение физической величины, характеризующей источник тока, ЭДС источника ε запишется как:

ε=Aq (1), где значение q подразумевает положительный заряд. Его движение происходит по замкнутому контуру. ЭДС – это не сила. Единица измерения ε=В.

Природа сторонних сил различна. В гальваническом элементе они являются результатом электрохимических процессов. В машине с постоянным током такой силой является сила Лоренца.

Основные характеристики электрического тока

Условно принято считать направление тока за направление движения положительных частиц. Отсюда следует, что направление тока в металлах характеризуется противоположным направлением относительно направления движения частиц.

Электрический ток обладает силой тока.

Определение 3

Сила тока I – скалярная величина, равняется производной от заряда q по времени для тока, который проходит через поверхность S:

I=dqdt (2).

Ток может быть постоянным и переменным. При неизменной силе тока  с его направлением по времени ток называют постоянным, а выражение силы тока для него примет вид:

I=qt (3), где сила тока рассматривается в качестве заряда, проходящего через поверхность S в единицу времени.

По системе СИ основная единица измерения силы тока – Ампер (А).

1 A=1 Кл1 с.

Определение 4

Плотность – это векторная локальная характеристика. Вектор плотности тока j→способен показывать, каким образом распределяется ток по сечению S. Его направление идет в сторону, куда движутся положительные заряды.

Значение вектора плотности тока по модулю равно:

j=dIdS’ (4), где dS’ является проекцией элементарной поверхности dS на плоскость, перпендикулярную вектору плотности тока, dI – элементом силы, которая идет через поверхности dS и dS’.

Представление плотности в металле возможно по формуле:

j→=-n0qeυ→ (5), где n0 обозначается концентрацией электронов проводимости, qe=1,6·10-19 Кл  – зарядом электрона, υ→ – средней скоростью упорядоченного движения электронов. Если значение плотностей тока максимальное, то

υ→=10-4 мс.

Нужна помощь преподавателя?

Опиши задание — и наши эксперты тебе помогут!

Описать задание

Закон сохранения заряда

Рисунок 1

Основным физическим законом считается закон сохранения электрического заряда. При выборе произвольной замкнутой поверхности S, изображенной на рисунке 1, ограничивающей объем V количество выходящего электричества в единицу времени (1 секунду) из объема V можно определить по формуле ∮sjndS. Такое же количество электричества выражается через заряд -∂q∂t, тогда получаем:

∂q∂t=-∮SjndS (6), где jn считается проекцией вектора плотности на направление нормали к элементу поверхности dS, при этом:

jn=jcos a (7), где a является углом между направлением нормали к dS и вектором плотности тока. Уравнение (6) показывает частое употребление производной для того, чтобы сделать акцент на неподвижности поверхности S.

Выражение (6) считается законом сохранения электрического заряда в макроскопической электродинамике. Если ток постоянен во времени, тогда запись этого закона примет вид:

∮SjndS=0 (8).

Пример 1

Найти формулу для того, чтобы рассчитать конвекционный ток при его возникновении в длинном цилиндре с радиусом сечения R и наличием его равномерной скорости движения υ, который заряжен по поверхности равномерно. Значение напряженности поля у поверхности цилиндра равняется E. Направление скорости движения вдоль оси цилиндра.

Решение

Основой решения задачи берется определение силы тока в виде:

I=dqdt (1.1).

Из формулы (1.1) следует, что возможно нахождение элемента заряда, располагающегося на поверхности цилиндра.

Напряженность поля равномерно заряженного цилиндра на его поверхности находится по выражению:

E=σε0 (1.2), где σ является поверхностной плотностью заряда, ε0=8,85·10-12 КлН·м2. Выразим σ из (1.2), тогда:

σ=E·ε0 (1.3).

Связь поверхностной плотности заряда с элементарным зарядом выражается при помощи формулы:

dqdS=σ (1.4).

Используя (1.3), (1.4), имеем:

dq=E·e0dS (1.5).

Выражение элемента поверхности цилиндра идет через его параметры:

dS=2π ·Rdh (1.6), где dh является элементом высоты цилиндра. Запись элемента заряда поверхности цилиндра примет вид:

dq=E·ε0·2h·Rdh (1.7).

Произведем подстановку из (1.7) в (1.1):

I=d(E·ε0·2π·Rdh)dt=2πRε0Edhdt (1.8).

Движение цилиндра идет вдоль оси, тогда запишем:

dhdt=υ (1.9).

Получим:

I=2πRε0Eυ.

Ответ: конвективный ток I=2πRε0Eυ.

Пример 2

Изменение тока в проводнике происходит согласно закону I=1+3t. Определить значение заряда, проходящего через поперечное сечение проводника, за время t, изменяющегося от t1=3 с до t2=7 c. Каким должен быть постоянный электрический ток, чтобы за аналогичное время происходило то же значение заряда?

Решение

Основа решения задачи – выражение, связывающее силу тока и заряд, проходящий через поперечное сечение проводника:

I=dqdt (2.1).

Формула (2.1) показывает, что нахождение количества заряда, проходящего через поперечное сечение проводника за время от t1 до t2 возможно таким образом:

q=∫t1t2Idt (2.2).

Произведем подстановку имеющегося по условию закона в (2.2) для получения:

q=∫t1t2(1+3t)dt=∫t1t2dt+∫t1t23tdt=t2-t1+3·t22t1t2=(t2-t1)+32t22-t12 (2.3).

Вычислим заряд:

q=7-3+32(72-32)=4+32·40=64 (Кл).

Чтобы определить постоянный ток для получения силы используется формула:

Iconst=qt (2.3), где t считается временем, за которое поперечное сечение проводника пройдет заряд q.

Тогда время протекания заряда равняется:

t=t2-t1 (2.4).

Выражение (2.3) примет вид:

Iconst=qt2-t1 (2.5).

Произведем подстановку и вычислим:

Iconst=647-3=644=16 (A).

Ответ: q=64 Кл. Iconst=16 А..

Определение постоянного и переменного электрического тока

Электричество – это тип энергии, передаваемый движением электронов через проводящий материал. Например, металлы представляют собой материалы с высокой электропроводностью и позволяют легко перемещать электроны. Внутри проводящего материала электроны могут двигаться в одном или нескольких направлениях.

Электрический ток

Понятие о постоянном и переменном токе

Что такое постоянный ток, определяется из характера движения электрозарядов. Аналогично можно установить, что такое переменный ток.

  1. Когда поток электрозарядов задан в одном направлении, он считается постоянным током;
  2. Когда электронный поток меняет направление и интенсивность во времени, он называется переменным током. Причем изменения идут циклически, по синусоидальному закону.

Большинство современных электросетей используют переменный электрический ток, производящийся на электростанциях соответствующими генераторами.

Графики постоянного и переменного токов

Постоянный ток (DC) генерируется батареями, топливными элементами и фотоэлектрическими модулями. Существуют и генераторы постоянного тока. Другое его получение – преобразование из однофазного и трехфазного переменного тока (АС) с помощью выпрямительных устройств.

В обратном случае АС может быть получен из DC, используя инверторы, хотя технология здесь несколько сложнее.

История

В природе электричество встречается относительно редко: оно генерируется только несколькими животными и существует в некоторых природных явлениях. В поисках искусственной генерации потока электронов ученые поняли, что можно заставить электроны проходить через металлическую проволоку или другой проводящий материал, но только в одном направлении, так как они отталкиваются от одного полюса и притягиваются к другому. Так родились батареи и генераторы постоянного тока. Изобретение приписывается, в основном, Томасу Эдисону.

В конце 19-го века другой известный ученый, Никола Тесла, разрабатывал способы получения переменного тока. Основными причинами работ в этой области явились обнаруженные недостатки постоянного тока при передаче электроэнергии на большие дистанции. Оказалось, что для переменного тока гораздо проще повысить напряжение передающих линий, тем самым уменьшив потери и получив возможность транспортировки больших объемов электрической энергии, а эффективно повысить напряжение на линиях с постоянным током в те времена было неосуществимо.

Для получения переменного тока Тесла использовал вращающееся магнитное поле. Если МП изменяет направленность, направление электронного потока также варьируется, и генерируется переменный ток.

Изменение направления в электронном потоке осуществляется очень быстро, много раз в секунду. Измерения частоты производятся в герцах (равных циклам в секунду). Таким образом, переменный ток частоты 50 Гц можно представить, как выполнение 50 циклов в секунду. В каждом цикле электроны изменяют направление и возвращаются к первоначальному, поэтому поток электронов изменяет направленность 100 раз в секунду.

Сравнительные характеристики постоянного и переменного токов

Разница между двумя видами токов заключена в их природе и вытекающих из этого свойствах.

Отличие постоянного тока от переменного:

  1. При переменном токе изменяется направленность и интенсивность электронного потока, при постоянном – она неизменна;
  2. Частота постоянного тока не может существовать. Это понятие применимо только для переменного тока;
  3. Полюсы (плюс и минус) всегда одинаковы в электроцепи постоянного тока. В электроцепи переменного тока положительные и отрицательные полюса меняются с периодическими интервалами;
  4. При передаче переменного тока напряжение легко преобразуется и транспортируется с приемлемым уровнем потерь.

Изменение полярности подключения DC может привести к необратимому повреждению устройств. Чтобы этого избежать, на оборудовании обычно ставятся обозначения полюсов. Аналогично контакты отличаются традиционным использованием металлической пружины для отрицательного полюса и пластины – для положительного. В устройствах с перезаряжаемыми батареями трансформатор-выпрямитель имеет выход, так что соединение выполняется только одним способом, что предотвращает инверсию полярности.

Обозначение полярности на аккумуляторе

В крупномасштабных установках, например, на телефонных станциях и другом телекоммуникационном оборудовании, где имеется централизованное распределение постоянного тока, используются специальные соединительные и защитные элементы,

Постоянный и переменный ток имеют свои достоинства и недостатки, отражающиеся на области их применения. По преимуществу широта использования переменного тока объясняется легкостью его преобразования.

Различия при транспортировке

Когда ток течет, часть энергии электронов преобразуется в тепло, благодаря активному сопротивлению проводов. Электрические нагреватели тоже основаны на этом эффекте. В конце линии меньше энергии передается потребителю. Рассеиваемые мощности называются потерями. Для уменьшения потерь применяется повышение напряжения при транспортировке. Эти физические зависимости применимы и к постоянному, и к переменному току, однако при реализации схем передачи возникают различия.

Достоинства и недостатки переменного тока

При начале строительства передающих электросетей использование трансформаторов было единственной возможностью получать высокие напряжения и затем снижать их до нужного уровня при распределении к потребителям. Такая технология называлась трансформаторной, и до сих пор структура транспортировки электроэнергии не изменилась. Почти повсеместно используется переменный ток, который представляет собой трехфазные системы.

ЛЭП переменного тока

Позже стали конструироваться и линии постоянного тока, которые последние годы используются все шире. Возросший интерес к их применению объясняется существенными недостатками систем переменного тока: в длинных линиях потери электроэнергии значительны. Причинами их являются наличие емкостного и индуктивного сопротивлений.

  1. При быстрой смене направления потока электронов наблюдается похожий на перезарядку конденсаторов эффект. Возникают дополнительные емкостные токи. Особенно это сказывается на наземных и подводных кабелях, изолирующий слой которых обладает высоким конденсаторным эффектом;
  2. Индуктивное сопротивление линий появляется потому, что электрические токи генерируют магнитные поля, меняющиеся с частотой тока. Появляются индуктивные токи.

Важно! Оба вида реактивных сопротивлений возрастают с увеличением протяженности линий.

Достоинства переменного тока:

  • легкая трансформация напряжения;
  • возможность комбинирования различных систем передачи;
  • возможность использования общесистемной частоты.

Недостатки переменного тока:

  • необходимость компенсации реактивной мощности при транспортировке на значительные расстояния;
  • сравнительно высокие потери.

Достоинства и недостатки постоянного тока

В первую очередь, чем отличается переменный ток от постоянного, – это присутствием источников потерь на реактивную энергию. Однако постоянный электрический ток предполагает потери на нагрев. Точное их определение зависит от технологии и уровня напряжения. Для высоких напряжений – около 3% на 1000 км.

Другим источником потерь в системах электропередачи на постоянном токе служат подстанции для преобразования переменного тока в постоянный, и наоборот. Суммарные потери намного ниже, чем для переменного тока, но существенными являются материальные затраты на строительство этих подстанций.

Оборудование для высоковольтной ЛЭП постоянного тока

Важно! Для повышения рентабельности линий электропередачи на постоянном токе применяются ЛЭП большой длины.

Техническое развитие в последнее время получила передача электроэнергии на постоянном токе, благодаря разработке новых электронных компонентов для создания высоких уровней напряжения постоянного тока – высокопроизводительных тиристоров или биполярных транзисторов.

Интересно. Сегодня возможны системы передачи постоянного тока с напряжением до 800 кВ и пропускной способностью до 8000 мВт на расстояние более 2000 км.

Преимущества высоковольтных ЛЭП постоянного тока:

  • возможность передачи мощности по подводным, наземным и подземным кабельным линиям на большие расстояния;
  • нет потерь из-за реактивной мощности;
  • лучшее использование изоляции кабелей.

Недостатки высоковольтных ЛЭП постоянного тока:

  • недостаточно быстрая коммутация существующих каналов постоянного тока;
  • мало стандартизированной электротехники;
  • не развиты распределительные сети передачи электроэнергии, транспортировка ведется от пункта до пункта.

Другие варианты применения постоянного и переменного тока

  1. DC идеально подходит для зарядки аккумуляторов и батарей элементов. Им нужно такое питание, потому что зарядная мощность всегда должна идти в одном направлении. Соответственно, устройства, работающие от аккумуляторов, также нуждаются в DC, например, фонарик или ноутбук;
  2. Телевидение, радио, компьютерная техника используют DC;
  3. Используемые в промышленности и в быту электродвигатели работают как на АС, так и на DC. То же относится к плитам, утюгам, чайникам и лампам накаливания;
  4. DC нужен для установок электролиза, где важно наличие неизменных полюсов. Только иногда полярность соблюдать не обязательно, в частности при электролизе газов. Тогда может применяться переменный электроток;
  5. Около половины мировых контактных сетей железнодорожного транспорта используют DC. В начале развития электрифицированных железных дорог были попытки применения трехфазных двигателей, но создание контактной сети для них столкнулось с проблемами. На DC работает городской электротранспорт: трамваи, троллейбусы, метро. Другой способ устройства железнодорожных контактных сетей – применение одной фазы переменного тока;

Контактная сеть железных дорог

  1. Для измерения токов, напряжений и мощности существуют приборы. Есть работающие только на DC, как магнитоэлектрические амперметры, а также использующие только АС, как индукционные счетчики. Часто используют универсальную измерительную технику.

Оба вида тока востребованы и применяются в различных областях. Какой из них использовать, зависит от принципа работы электрооборудования и приборов.

Видео

Оцените статью:

2. Постоянный электрический ток. Электричество и магнетизм. Физика. Курс лекций

2.1. Плотность тока носителей заряда разных знаков

2.2. ЭДС. Источник тока. Напряжение

2.3. Законы Ома в интегральной форме

2.3.1. Закон Ома в дифференциальной форме

2.4. Закон Джоуля-Ленца

2.5. Законы Кирхгофа

2.6. Эмиссия электронов с поверхности

2.6.1. Работа выхода

2.6.2. Способы выбивания (отрыва) электронов с поверхности

2.6.3. Электрический ток в вакууме

2.7. Заряженная частица в плоском конденсаторе

Электрический ток — направленное движение зарядов.

Направление тока — направление движения «+» зарядов. Так исторически принято, хотя основными носителями заряда в подавляющем большинстве случаев являются электроны, т.е. отрицательно заряженные частицы.

Условия возникновения электрического тока:

1. наличие свободных носителей электрических зарядов. 2. электрическое поле (внешнее).

Характеристики электрического поля:

Сила тока — количество заряда, протекающего по проводнику в единицу времени. Для постоянного тока:

.

Если количество заряда меняется со временем, то:

.

Плотность тока — численно равна величине тока, протекающего через единичную площадку, расположенную перпендикулярно направлению движения заряда.

.

Если сила тока величина скалярная, то плотность тока – вектор, направленный вдоль нормали к поверхности, через которую протекает заряд. Если поперечное сечение проводника, по которому протекает ток, неоднородно, тогда плотность тока в разных частях проводника выражается дифференцированием, т.е. величина силы тока есть поток векторов j, через поперечное сечение проводника (см. т.Гаусса).

— самостоятельного наименования не имеет.

Электропроводность — физическая величина, количественно характеризующая способность тела пропускать электрический ток под действием электрического поля (- электропроводность).

Величина, обратная электропроводности, называется сопротивлением.

.

Сопротивление протеканию тока есть величина, характеризующаяся структурными и химическими особенностями среды, по которой протекает заряд. Структурные особенности — взаимное расположение атомов в проводнике, химическая особенность — разного рода молекулярная связь атомов и молекул вещества.

Эти особенности, как правило табличные, и называются удельным сопротивлением — сопротивлением проводника протеканию электрического тока телом с геометрическим размером ~1м3:

где ρ — удельное сопротивление, — длина, S — площадь поперечного сечения физического тела.

Поскольку сопротивление определяется особенностями строения проводника, то температура окружающей среды, искажающая состояние структуры химических связей атомов вещества, оказывает решающее влияние на это сопротивление. Из общих соображений можно сказать, что повышение температуры повышает сопротивление.

Rt=Ro+(1+a t) где Rо- сопротивление при комнатной температуре, t — температура в градусах Цельсия, α — температурный коэффициент сопротивления.

Изменение температуры на десятки градусов изменяет сопротивление на несколько процентов, на сотни градусов — на десятки процентов. (α ~ 10-3 К-1).

2.1. Плотность тока носителей заряда разных знаков

В общем случае для разных типов носителей заряда: где ρ = n· e, n — удельная концентрация зарядов, e — заряд электрона ( e=1,6·10-19 Кл ), ρ — объемная плотность заряда. — количество заряда в данном проводнике длиной l и поперечным сечением S.

Аналогичное математическое рассмотрение можно провести, как для “+” так и для “-” зарядов. Предполагается, что “+” и “-” заряды при протекании не взаимодействуют друг с другом, тогда общие потоки зарядов движутся навстречу друг другу и результирующий поток равен:

, если . Здесь скорости положительных (+) и отрицательных (-) зарядов, которые, как правило, не одинаковы. Итак, плотность потока зарядов противоположного знака численно равна сумме плотностей потоков отдельных зарядов

2.2. ЭДС. Источник тока. Напряжение

Чтобы в проводнике протекал постоянный электрический ток: 1) подают на один конец заряды, а на другом их снимают; и 2) нужны некоторые силы, чтобы заряд перемещался, т.е. нужны силы неэлектрического происхождения, их называют сторонние силы.

Сторонние силы не должны быть электрическими, а должны быть химическими, ядерными, механическими и т.д. для совершения работы по перемещению заряда по участку цепи. Участок цепи, в который включается источник сторонних сил обозначается двумя перпендикулярными линиями: тонкая длинная — источник «+» зарядов, толстая короткая — источник «-» .

Устройство, в котором возникают сторонние силы, называются источником тока. Если потенциалы φ1, φ2 в точках 1 и 2 создаются так же электрическими силами, тогда полная сила, вынуждающая заряды двигаться, F=Fстор+FK , а работа по перемещению заряда из точки (1) в точку (2) А12=Fr, если ток протекает в цепи постоянный:

А12=Fr=Fстор · r+FK· r =Eстор·qr+Eкул·qr

Введем понятие силовой характеристики сторонних сил, заставляющих заряды q двигаться, такое как, напряженность поля сторонних сил, тогда:

Fстор =Eстор·q

Зная связь между напряженностью и разностью потенциалов, можем записать, что:

Тогда полная работа:

А12=Eстор·qr + (φ12 )·q.

Разделив это уравнение на величину переносимого заряда q, получим: .

Это напряжение, получаемое на концах участка цепи 1-2, содержащего сторонние силы. Согласно определению силовой характеристики сторонних сил можно записать:

есть электродвижущая сила источника сторонних сил.

ЭДС (e) — электродвижущая сила источника сторонних сил; тогда выражение напряжения на концах участка цепи, содержащего сторонние силы, численно определяется с “+” , если э.д.с. помогает протеканию тока; и с “-” , если э.д.с. препятствует протеканию тока.

2.3. Законы Ома в интегральной форме

Закон Ома в интегральной форме подразумевает, что рассматривается полный ток, протекающий в цепи и величина тока со временем не меняется. Очевидно, что количество заряда, протекающее по проводнику, обратно пропорционально сопротивлению проводника. Количество заряда протекающее в проводнике, прямо пропорционально напряженности или разности потенциалов, создающих внешнее электрическое поле.

1) — закон Ома для участка цепи, не содержащего э.д.с.

Суммарное сопротивление проводников и элементов цепи без э.д.с. обозначается на схеме.

2) Если участок цепи включает в себя э.д.с, то собственное сопротивление источника тока выделяется и обозначается r.

Тогда закон Ома для участка цепи, содержащей э.д.с., будет иметь вид:

.

3) Если замкнутый участок цепи, содержит э.д.с., тогда φ1 = φ2, и получаем:

— закон Ома для замкнутого участка цепи, содержащего э.д.с.

В целом участок цепи, содержащей множество э.д.с. и разных деталей представлен законом Ома в виде:

.

Если при напряжении на концах участка цепи в 1В по цепи протекает ток в 1А, то говорят, что сопротивление цепи равно одному Ому.

Из закона Ома следует:

.

2.3.1. Закон Ома в дифференциальной форме

Сечение проводника или элементов цепи, как правило, неоднородно, и сопротивляемость в разных участках цепи протеканию тока также различная. Тогда разбивают участки цепи на элементы (дифференцируют) и определяют закон Ома в каждом отдельном участке.

— закон Ома, тогда для каждого участка цепи сечением ∆S и длиной ∆l можно записать закон Ома как: .

Учитывая, что для участка цепи

и , получим .

Это закон Ома в дифференциальной форме. Зная, что удельная электропроводность σ и удельное сопротивление ρ связаны, как:

, где

σ — удельная электропроводность,

ρ — удельная сопротивление,

— закон Ома в дифференциальной форме.

2.4. Закон Джоуля-Ленца

В интегральной форме

Закон Джоуля-Ленца касается закона сохранения энергии; если считать, что система электрической цепи замкнутая, то работа по перемещению заряда в проводнике, если сам проводник не перемещается в пространстве, полностью преобразуется в тепловую энергию Q на участке (1-2).

Учитывая, что q=I· t получаем:

Q=IU· t (1) (2)

(3)

Вид формулы для Q определяется условием задачи по определению выделившегося тепла. Формулы (1), (2), (3) есть закон Джоуля-Ленца в интегральной форме (определение полного тепла, выделившегося в цепи за все время протекания тока).

Тепловая мощность тока.

Для определения количества теплоты, выделившегося в единицу времени, вводят понятие тепловой мощности тока:

.

Единицей мощности тока считают 1Вт=1Дж/1с.

В дифференциальной форме

Если электрическая цепь состоит из элементов различного сопротивления и геометрии, то цепь разбивают на отдельные участки и определяют закон Джоуля-Ленца для каждого участка. Последовательно расписывая

Из закона Ома в дифференциальной форме следует:

, т.к.

Количество тепла, выделяемое в единице объема проводника за единицу времени равно квадрату плотности тока, умноженному на ρ, или квадрату напряженности электрического поля, деленному на ρ. Это закон Джоуля-Ленца в дифференциальной форме:

.

2.5. Законы Кирхгофа

I Закон Кирхгофа — закон токов (для узлов цепей).

В участке электрических цепей очень часто содержатся узлы, в которых сходятся множество элементов, проводящих ток.

Если цепь работающая, то по разным участкам будут протекать различные токи. По закону сохранения заряда, как материального объекта, можно предположить, что количество заряда, приходящего в узел, должно быть численно равно количеству заряда, выходящего из узла:

разделив на t получаем:

, т.е. по определению

Окончательно имеем:

Сумма электрических токов, сходящихся в узле работающих цепей, всегда равна нулю.

II Закон Кирхгофа — закон напряжений (закон замкнутых цепей).

Величина электрического тока в последовательных цепях есть величина постоянная и по закону сохранения заряда , а по закону Ома на каждом участке:

. Сложим левые и правые части уравнений:

.

Окончательно получаем .

В любом замкнутом контуре сумма падений напряжений на всех участках цепи равна алгебраической сумме э.д.с., включенных в цепь.

2.6. Эмиссия (испускание) электронов с поверхности

Так как любое вещество имеет в своем объеме свободные электроны, то любое внешнее электрическое воздействие на вещество может привести к отрыву электронов с поверхности вещества (эмиссия).

Итак, для того, чтобы удалить электрон с поверхности вещества, требуется совершить работу. Принципиально свободные электроны могут испускаться поверхностями любых веществ, где есть граница раздела (воздух-вода, дерево-вакуум).

Но наибольшее количество испускаемых электронов наблюдается у металлов в связи с наибольшим количеством свободных электронов у этого класса веществ. Эмиссия электронов характеризуется работой выхода — минимальной энергией, которую необходимо затратить для удаления электрона с поверхности твердого или жидкого вещества в вакуум.

2.6.1. Работа выхода

Энергетический разрыв между энергиями электронов в атоме и энергиями электронов в свободном состоянии (в кристалле) называется энергией отрыва электрона от атома. Значит энергетическое состояние свободного электрона больше, чем энергия электрона в атоме. Точно также для отрыва свободного электрона с поверхности вещества требуется совершить работу. Значит, энергетическое состояние электрона вне вещества выше, чем энергия электрона в кристалле.

Для чистых веществ работа выхода зависит только от особенностей атома вещества и взаимосвязей атомов между собой.

Для разных веществ Авых не превышает нескольких эВ, например:

Металл Pt W Mo Fe Mg Na
Авых (эВ) 5,29 4,5 4,27 4,36 3,45 2,27

2.6.2. Способы выбивания (отрыва) электронов с поверхности

Фотоэлектронная эмиссия — выбивание электронов с поверхности под действием электромагнитного излучения (свет — это часть диапазона электромагнитных волн).

И — источник электромагнитных волн (света).

а) окошко для света закрыто, тока нет, т.е. I=0;

б) окошко для света открыто, ток есть, то есть I≠0, т.к. свет падает на поверхность электрода, выбивает электроны, которые и создают ток между анодом и катодом.

Вторичная электронная эмиссия — испускание электронов с поверхности вещества под действием бомбардировки внешних электронов.

Если энергия внешних электронов достаточна для совершения работы выхода (отрыва) электронов с поверхности, то общий поток электронов между анодом и катодом возрастает.

Это устройство называют электронным умножителем.

Автоэлектронная эмиссия — вырывание электронов с поверхности вещества под действием внешнего электрического поля (холодная эмиссия).

Острие катода является концентратором электрического поля. При повышении напряжения между электродами возникает ситуация, когда энергия электрического поля превышает Авых электрона с поверхности.

Задавая напряжение .

Если — условие автоэлектронной эмиссии.

Термоэлектронная эмиссия — явление вырывания электрона с поверхности вещества под действием тепла. При этом тепло или энергия, подводимая к поверхности вещества, превышает работу выхода Q = I2· Rt Aвых. Это явление используется в работе электронно-лучевых трубок.

2.6.3. Электрический ток в вакууме

Электрод, на который подается “+” потенциал, называется анод, а “-” потенциал — катод. Эти электроды помещены в замкнутую вакуумированную среду, а все устройство называют вакуумным диодом.

Пропуская по катоду регулируемый электрический ток по закону Джоуля-Ленца мы вызываем его нагрев. В результате нагрева с поверхности катода испускаются термоэлектроны. Под действием электрического поля между катодом и анодом электроны летят на анод, цепь замыкается, приборы фиксируют наличие тока.

Анализ зависимости тока от напряжения называется вольтамперной характеристикой. ВАХ вакуумного диода имеет сложный характер насыщения.

Проанализируем характерные точки:

1) При отсутствии напряжения между анодом и катодом, электроны вылетают с катода хаотично и часть электронов может попасть на анод; эта величина тока очень мала, но физически имеет место.

2) При увеличении напряжения между анодом и катодом электроны, вылетающие с катода, вытягиваются электрическим полем к аноду и величина тока возрастает; зависимость тока от напряжения на этом участке происходит по закону Богуславского — Ленгмюра (закон 3/2): .

3) участок называется током насыщения; при дальнейшем увеличении напряжения между анодом и катодом наступает момент, когда все электроны, вылетающие с катода, вытягиваются электрическим полем на анод, и дальнейшее увеличение напряжения не приводит к увеличению тока, т.к. количество электронов, вылетающих с катода, ограничено.

4) для того, чтобы полностью подавить анодный ток, необходимо между электродами подать обратное напряжение, и величина напряжения, при котором анодный ток равен 0, называется Uзап — запирающим напряжением.

Поскольку электроны, вылетающие с поверхности, как правило, обладают кинетической энергией, то по данным точки (4) по закону сохранения энергии можно рассчитать скорость вылета электронов, если запирающее напряжение — несколько вольт:

Это среднее значение скорости электронов, летящих от катода к аноду. Величину тока насыщения вакуумного диода можно изменять, изменив нагрев катода, т.е. T3> T2> T1 и, соответственно, изменяется количество электронов, вылетающих с поверхности, как следствие, изменяется Iнас3> Iнас2> Iнас1 .

Зависимость тока насыщения от температуры — закон Риичардсона-Дэшмана и имеет вид:

2.7. Заряженная частица в плоском конденсаторе

Рассмотрим два случая поведения заряженной частицы в конденсаторе.

а) частица движется перпендикулярно пластинам.

Напишем уравнение для отдельного электрона. По закону сохранения энергии работа по переносу заряда от пластины до пластины:

.

б) частица движется параллельно пластинам.

Также рассмотрим действие поля конденсатора на электрон. По 2-му закону Ньютона сила Кулона вызывает ускорение в направлении, перпендикулярном пластинам, и отклоняет электрон к “+” пластине:

;

Зная, ;

Разложим скорость электрона на две составляющие: параллельную и перпендикулярную пластинам. — параллельна пластинам. Эта скорость не меняется, т.к. вдоль пластин нет силы, действующей на электрон. Перпендикулярная составляющая — , (если электрон влетел в конденсатор параллельно пластинам, ), определится в середине между обкладками как:

.

Тогда путь, пройденный электроном в направлении, перпендикулярном пластинам:

Тогда время пролета электрона в конденсаторе параллельно пластинам:

В результате этого анализа можно сказать, что электрон может выйти из конденсатора, если , а если , то электрон ударится об электрод, т.е. время пролета расстояния меньше времени, затраченного на прохождение пути .

в чем разница между ними

В электричестве есть два рода тока – постоянный и переменный. Устройства также требуют для питания один или другой вид тока. От этого зависит возможность их работы, а иногда и целостность после подключения к неправильному питанию. Чем отличается переменный ток от постоянного мы расскажем в этой статье, дав краткий ответ наиболее простыми словами.

Определение

Электрическим током называется направленное движение заряженных частиц. Так звучит определение из учебника по физике. Простыми словами можно перевести так, что у его составляющих всегда есть какое-то направление. Собственно, это направление и является определяющем в сегодняшнем разговоре.

Переменный ток (Alternative Current – AC) отличается от постоянного (Direct Current – DC) тем, что у последнего электроны (носители заряда) всегда движутся в одном направлении. Соответственно отличием переменного тока является то, что направление движения и его сила зависят от времени. Например, в розетке направление и величина напряжения, соответственно и сила тока, изменяется по синусоидальному закону с частотой в 50 Гц (50 раз за секунду изменяется полярность между проводами).

Для так сказать чайников в электрике изобразим это на графике, где по вертикальной оси изображена полярность и напряжение, а по горизонтальной время:

Красной линией изображено постоянное напряжение, оно остаётся неизменным с течением времени, разве что изменяется при коммутации мощной нагрузки или КЗ. Зелеными волнами показан синусоидальный ток. Вы можете видеть, что он протекает то в одну, то в другую сторону, в отличие от постоянного тока, где электроны всегда протекают от минуса к плюсу, а направлением движения электрического тока выбран путь от плюса к минусу.

Если сказать по-простому, то разницей в этих двух примерах является то, что у постоянки всегда плюс и минус находятся на одних и тех же проводах. Если говорить о переменном, то в электроснабжении используют понятия фазы и нуля. Если рассматривать по аналогии с постоянкой, то фаза и ноль являются плюсом и минусом, только полярность меняется 50 раз в секунду (в США и ряде других стран 60 раз в секунду, а в самолётах более 400 раз).

Происхождение

Разница между AC и DC заключается в их происхождении. Постоянный ток можно получить из гальванических элементов, например, батареек и аккумуляторов.

Также его можно получить с помощью динамомашины – это устаревшее название генератора постоянного тока. Кстати с их помощью генерировалась энергия для первых электросетей. Мы об этом говорили в статье об открытиях Николы Тесла, в заметках о войне идей между Теслой и Эдисоном. Позже так называли небольшие генераторы для питания велосипедных фар.

Переменный ток добывают также с помощью генераторов, в наше время в основном трёхфазных.

Также и то и другое напряжение можно получить с помощью полупроводниковых преобразователей и выпрямителей. Так вы можете выпрямить переменный ток или получить его же, преобразовав постоянный.

Формулы для расчета постоянного тока

Разницей между переменкой и постоянкой являются и формулы для расчетов процессов, происходящих в цепи. Так сопротивление рассчитываются по Закону Ома для участка цепи или для полной цепи:

E=I/R

E=I/(R+r)

Мощность также просто рассчитываются:

P=UI

Формулы для расчета переменного тока

В расчётах цепей переменного тока разница в формулах обусловлена отличием процессов, протекающих в емкостях и индуктивностях. Тогда формула закона Ома будет для активного сопротивления:

Для ёмкости:

Для индуктивности:

Здесь 1/wC и wL – емкостное и индуктивное реактивные сопротивления, а w – угловая частота, она равна 2пиF.

Для цепи с ёмкостью и индуктивностью:

wL-1/wC – это реактивное сопротивление, оно обозначается как Z.

На видео ниже более подробно рассказывается, в чем отличие переменного тока от постоянного:

Материалы по теме:

Что такое ток: основные понятия

Что же такое ток и напряжение на пальцах

Что называют силой тока? Такой вопрос не раз и не два возникал у нас в процессе обсуждения различных вопросов. Поэтому мы решили разобраться с ним более подробно, и постараемся сделать это максимально доступным языком без огромного количества формул и непонятных терминов.

Что такое электрический ток

Итак, что называется электрическим током? Это направленный поток заряженных частиц. Но что это за частицы, с чего это вдруг они двигаются, и куда? Это все не очень понятно. Поэтому давайте разберемся в этом вопросе подробнее.

Носители электрического заряда в различных материалах

  • Начнем с вопроса про заряженные частицы, которые, по сути, являются носителями электрического тока. В разных веществах они разные. Например, что представляет собой электрический ток в металлах? Это электроны. В газах — электроны и ионы; в полупроводниках – дырки; а в электролитах — это катионы и анионы.

Строение атома

  • Эти частицы имеют определенный заряд. Он может быть положительным или отрицательным. Определение положительного и отрицательного заряда дано условно. Частицы, имеющие одинаковый заряд, отталкиваются, а разноименный — притягиваются.

Электрический ток

  • Исходя из этого, получается логичным, что движение будет происходить от положительного полюса к отрицательному. И чем большее количество заряженных частиц имеется на одном заряженном полюсе, тем большее их количество будет перемещаться к полюсу с другим знаком.
  • Но все это глубокая теория, поэтому давайте возьмем конкретный пример. Допустим, у нас имеется розетка, к которой не подключено ни одного прибора. Есть ли там ток?
  • Для ответа на этот вопрос нам необходимо знать, что такое напряжение и ток. Дабы это было понятнее, давайте разберем это на примере трубы с водой. Если говорить упрощенно, то труба — это наш провод. Сечение этой трубы — это напряжение электрической сети, а скорость потока — это и есть наш электрический ток.
  • Возвращаемся к нашей розетке. Если проводить аналогию с трубой, то розетка без подключенных к ней электроприборов, это труба, закрытая вентилем. То есть электрического тока там нет.

Электрический ток появится тогда, когда появится нагрузка, а для этого нужно вставить вилку в розетку

  • Но зато там есть напряжение. И если в трубе, для того чтоб появился поток, необходимо открыть вентиль, то чтобы создать электрический ток в проводнике, надо подключить нагрузку. Сделать это можно путем включения вилки в розетку.
  • Конечно, это весьма упрощенное представление вопроса, и некоторые профессионалы будут меня хаять и указывать на неточности. Но оно дает представление о том, что называют электрическим током.

Постоянный и переменный ток

Виды электрического тока

Следующим вопросом, в котором мы предлагаем разобраться – это: что такое переменный ток и постоянный ток. Ведь многие не совсем правильно понимают эти понятия.

Постоянный ток

Постоянным называется ток, который в течение времени не изменяет своей величине и направлению. Достаточно часто к постоянному еще относят пульсирующий ток, но давайте обо всем по порядку.

Постоянный ток

  • Постоянный ток характеризуется тем, что одинаковое количество электрических зарядов постоянно сменяет друг друга в одном направлении. Направление — это от одного полюса, к другому.
  • Получается, что проводник всегда имеет либо положительный, либо отрицательный заряд. И в течение времени это неизменно.

Обратите внимание! При определении направления постоянного тока, могут быть несогласности. Если ток образуется движением положительно заряженных частиц, то его направление соответствует движению частиц. Если же ток образован движением отрицательно заряженных частиц, то его направление принято считать противоположным движению частиц.

Виды пульсирующего тока

  • Но под понятие, что такое постоянный ток достаточно часто относят и так называемый пульсирующий ток. От постоянного он отличается только тем, что его значение в течение времени изменяется, но при этом он не меняет своего знака.
  • Допустим, мы имеем ток в 5А. Для постоянного тока эта величина будет неизменной в течении всего периода времени. Для пульсирующего тока, в один отрезок времени она будет 5, в другой 4, а в третий 4,5. Но при этом он ни в коем случае не снижается ниже нуля, и не меняет своего знака.

Вариант преобразованного из переменного, постоянного пульсирующего тока

  • Такой пульсирующий ток очень распространен при преобразовании переменного тока в постоянный. Именно такой пульсирующий ток выдает ваш инвертор или диодный мост в электронике.
  • Одним из главных преимуществ постоянного тока является то, что его можно накапливать. Сделать это можно своими руками, при помощи аккумуляторных батарей или конденсаторов.

Переменный ток

Чтобы понять, что такое переменный ток, нам необходимо представить себе синусоиду. Именно эта плоская кривая лучше всего характеризует изменение постоянного тока, и является стандартом.

Синусоида переменного тока

Как и синусоида, переменный ток с постоянной частотой меняет свою полярность. В один период времени он положительный, а в другой период времени он отрицательный.

 

На фото основные параметры синусоиды

Поэтому, непосредственно в проводнике передвижения, носителей заряда, как такового, нет. Дабы понять это, представьте себе волну, набегающую на берег. Она движется в одну сторону, а затем — в обратную. В итоге, вода вроде движется, но остается на месте.

Частота переменного тока

Исходя из этого, для переменного тока очень важным фактором становится его скорость изменения полярности. Этот фактор называют частотой.

Чем выше эта частота, тем чаще за секунду меняется полярность переменного тока. В нашей стране для этого значения есть стандарт – он равен 50Гц.

То есть, переменный ток меняет свое значение от крайнего положительного, до крайнего отрицательного 50 раз в секунду.

Формула частоты переменного тока

Но существует не только переменный ток частотой в 50Гц. Многое оборудование работает на переменном токе отличных частот.

Ведь за счет изменения частоты переменного тока, можно изменять скорость вращения двигателей.

Можно так же получать более высокие показатели обработки данных – как например в чипсетах ваших компьютеров, и многое другое.

Обратите внимание! Наглядно увидеть, что такое переменный и постоянный ток, можно на примере обычной лампочки. Особенно хорошо это видно на некачественных диодных лампах, но присмотревшись, можно увидеть и на обычной лампе накаливания. При работе на постоянном токе они горят ровным светом, а при работе на переменном токе едва заметно мерцают.

Что такое мощность и плотность тока?

Ну вот, мы выяснили, что такое ток постоянный, а что такое переменный. Но у вас наверняка осталось еще масса вопросов. Их-то мы и постараемся рассмотреть в этом разделе нашей статьи.

Из этого видео Вы подробнее сможете узнать о том, что же такое мощность.

  • И первым из этих вопросов будет: что такое напряжение электрического тока? Напряжением называется разность потенциалов между двумя точками.

Что является электрическим напряжением

  • Сразу возникает вопрос, а что такое потенциал? Сейчас меня вновь будут хаять профессионалы, но скажем так: это избыток заряженных частиц. То есть, имеется одна точка, в которой избыток заряженных частиц — и есть вторая точка, где этих заряженных частиц или больше, или меньше. Вот эта разница и называется напряжением. Измеряется она в вольтах (В).

Напряжение в розетке

  • В качестве примера возьмем обычную розетку. Все вы наверняка знаете, что ее напряжение составляет 220В. В розетке у нас имеется два провода, и напряжение в 220В обозначает, что потенциал одного провода больше чем потенциал второго провода как раз на эти 220В.
  • Понимание понятия напряжения нам необходимо для того, чтоб понять, что такое мощность электрического тока. Хотя с профессиональной точки зрения, это высказывание не совсем верное. Электрический ток не обладает мощностью, но является ее производной.

Плотность электрического тока в проводнике

  • Дабы понять этот момент, давайте вновь вернемся к нашей аналогии с водяной трубой. Как вы помните сечение этой трубы — это напряжение, а скорость потока в трубе — это ток. Так вот: мощность — это то количество воды, которое протекает через эту трубу.
  • Логично предположить, что при равных сечениях, то есть напряжениях — чем сильнее поток, то есть электрический ток, тем больший поток воды переместиться через трубу. Соответственно, тем большая мощность передастся потребителю.
  • Но если в аналогии с водой мы через трубу определенного сечения можем передать строго определенное количество воды, так как вода не сжимается, то с электрическим током все не так. Через любой проводник мы теоретически можем передать любой ток. Но практически, проводник небольшого сечения при высокой плотности тока просто перегорит.

Формула плотности тока

  • В связи с этим, нам необходимо разобраться с тем, что такое плотность тока. Грубо говоря — это то количество электронов, которое перемещается через определенное сечение проводника за единицу времени.
  • Это число должно быть оптимальным. Ведь если мы возьмем проводник большого сечения, и будем передавать через него небольшой ток, то цена такой электроустановки будет велика. В то же время, если мы возьмем проводник небольшого сечения, то из-за высокой плотности тока он будет перегреваться и быстро перегорит.
  • В связи с этим, в ПУЭ есть соответствующий раздел, который позволяет выбрать проводники исходя из экономической плотности тока.

Таблица выбора проводников по экономической плотности тока

  • Но вернемся к понятию, что такое мощность тока? Как мы поняли по нашей аналогии, при одинаковом сечении трубы передаваемая мощность зависит только от силы тока. Но если сечение нашей трубы увеличить, то есть увеличить напряжение, в этом случае, при одинаковых значениях скорости потока, будут передаваться совершенно разные объемы воды. То же самое и в электрике.

Передача мощностей через лини разных напряжений и видов электрического тока

  • Чем выше напряжение, тем меньший ток необходим для передачи одинаковой мощности. Именно поэтому, для передачи на большие расстояния больших мощностей используют высоковольтные линии электропередач.

Ведь линия сечением провода в 120 мм2 на напряжение в 330кВ, способна передать в разы большую мощность в сравнении с линией такого же сечения, но напряжением в 35кВ. Хотя то, что называется силой тока, в них будет одинаковой.

Способы передачи электрического тока

Что такое ток и напряжение мы разобрались. Пришла пора разобраться со способами распределения электрического тока. Это позволит в дальнейшем более уверено чувствовать себя в общении с электроприборами.

Постоянный ток

Как мы уже говорили, ток может быть переменным и постоянным. В промышленности, и у вас в розетках используется переменный ток. Он более распространен, так как его легче передавать по проводам. Дело в том, что изменять напряжение постоянного тока достаточно сложно и дорогостояще, а изменять напряжение переменного тока можно при помощи обыкновенных трансформаторов.

Обратите внимание! Ни один трансформатор переменного тока не будет работать на постоянном токе. Так как свойства, которые он использует, присущи только переменному току.

Аккумуляторная батарея

  • Но это совсем не обозначает, что постоянный ток нигде не используется. Он обладает другим полезным свойством, которое не присуще переменному. Его можно накапливать и хранить.
  • В связи с этим, постоянный ток используют во всех портативных электроприборах, в железнодорожном транспорте, а также на некоторых промышленных объектах где необходимо сохранить работоспособность даже после полного прекращения электроснабжения.

Промышленная аккумуляторная батарея

  • Самым распространенным способом хранения электрической энергии, являются аккумуляторные батареи. Они обладают специальными химическими свойствами, позволяющими накапливать, а затем при необходимости отдавать постоянный ток.
  • Каждый аккумулятор обладает строго ограниченным объемом накапливаемой энергии. Ее называют емкостью батареи, и отчасти она определяется пусковым током аккумулятора.
  • Что такое пусковой ток аккумулятора? Это то количество энергии, которое аккумулятор способен отдать в самый первоначальный момент подключения нагрузки. Дело в том, что в зависимости от физико-химических свойств, аккумуляторы отличаются по способу отдачи накопленной энергии.

Графики разряда аккумуляторной батареи

  • Одни могут отдать сразу и много. Из-за этого они, понятное дело, быстро разрядятся. А вторые отдают долго, но по чуть-чуть. Кроме того, важным аспектом аккумулятора является возможность поддержания напряжения.
  • Дело в том, что как говорит инструкция, у одних аккумуляторов по мере отдачи емкости, плавно снижается и их напряжение. А другие аккумуляторы способны отдать практически всю емкость с одинаковым напряжением. Исходя из этих основных свойств, и выбирают эти хранилища для электроэнергии.
  • Для передачи постоянного тока, во всех случаях используется два провода. Это положительная и отрицательная жила. Красного и синего цвета.

Переменный ток

А вот с переменным током все намного сложнее. Он может передаваться по одному, двум, трем или четырем проводам. Чтоб объяснить это, нам необходимо разобраться с вопросом: что такое трехфазный ток?

  • Переменный ток у нас вырабатывается генератором. Обычно почти все их них имеют трёхфазную структуру. Это значит, что генератор имеет три вывода и в каждый из этих выводов выдается электрический ток, отличающийся от предыдущих на угол в 120⁰.

Синусоиды трехфазной сети переменного тока

  • Дабы это понять, давайте вспомним нашу синусоиду, которая является образцом для описания переменного тока, и согласно законам которой он изменяется. Возьмем три фазы – «А», «В» и «С», и возьмем определенную точку во времени. В этой точке синусоида фазы «А» находится в нулевой точке, синусоида фазы «В» находится в крайней положительной точке, а синусоида фазы «С» — в крайней отрицательной точке.
  • Каждую последующую единицу времени переменный ток в этих фазах будет изменяться, но синхронно. То есть, через определенное время, в фазе «А» будет отрицательный максимум. В фазе «В» будет ноль, а в фазе «С» — положительный максимум. А еще через некоторое время, они вновь сменятся.

Фазные и линейные напряжения трехфазной сети

  • В итоге получается, что каждая из этих фаз имеет собственный потенциал, отличный от потенциала соседней фазы. Поэтому между ними обязательно должно быть что-то, что не проводит электрический ток.
  • Такая разность потенциалов между двумя фазами называется линейным напряжением. Кроме того, они имеют разность потенциалов относительно земли – это напряжение называется фазным.
  • И вот, если линейное напряжение между этими фазами составляет 380В, то фазное напряжение равно 220В. Оно отличается на значение в √3. Это правило действует всегда и для любых напряжений.

Величины фазных и линейных напряжений

  • Исходя из этого, если нам необходимо напряжение в 220В, то можно взять один фазный провод, и провод, жестко подключенный к земле. И у нас получится однофазная сеть 220В. Если нам необходима сеть 380В, то мы можем взять только 2 любые фазы, и подключить какой-то нагревательный прибор как на видео.

Цветовое обозначение проводников трехфазной сети в разных странах мира

Но в большинстве случаев, используются все три фазы. Все мощные потребители подключаются именно к трехфазной сети.

Вывод

Что такое индукционный ток, емкостной ток, пусковой ток, ток холостого хода, токи обратной последовательности, блуждающие токи и многое другое, мы просто не можем рассмотреть в рамках одной статьи.

Ведь вопрос электрического тока достаточно объемен, и для его рассмотрения создана целая наука электротехника. Но мы очень надеемся, что смогли объяснить доступным языком основные аспекты данного вопроса, и теперь электрический ток не будет для вас чем-то страшным и непонятным.

Постоянный электрический ток

Эти заряженные частицы в теории часто называют носителями тока. В проводниках и полупроводниках носителями тока являются электроны, в электролитах заряженные ионы. В газах носителями заряда могут быть и электроны и ионы. В металлах, например, могут перемещаться только электроны. Следовательно, электрический ток в них — есть движение электронов проводимости. Надо отметить, что результат прохождения электрического тока в металлах и электропроводящих растворах существенно отличается. В металлах не происходит химических процессов при прохождении тока. Тогда как в электролитах под воздействием тока идет выделение ионов вещества на электродах (явление электролиза). Различие в результатах действия тока объясняется тем, что носители зарядов в металле и электролите принципиально различны. В металлах — это свободные электроны, которые отделились от атомов, в растворах — это ионы, то есть атомы или их группы, которые имею заряд.

Так, первым необходимым условием существования электрического тока, в каком — либо веществе является наличие носителей тока.

Для того чтобы заряды находились в равновесии необходимо, чтобы разность потенциалов между любыми точками проводника была равна нулю. В том случае, если это условие нарушается, то равновесия нет, тогда заряд перемещается. Следовательно, вторым необходимым условием существования электрического тока в проводнике является создание напряжения между некоторыми точками.

Упорядоченное движение свободных зарядов, которое возникает в проводнике как результат воздействия электрического поля, называют током проводимости.

Однако отметим, что упорядоченное движение заряженных частиц возможно в том случае, если заряженный проводник или диэлектрик перемещать в пространстве. Подобный электрический ток называют конвекционным.

Механизм осуществления постоянного тока

Для того чтобы ток в проводнике шел постоянно, необходимо, чтобы к проводнику (или совокупности проводников — цепь проводников) было присоединено какое — либо устройство, в котором постоянно происходил процесс разделения электрических зарядов и тем самым поддерживалось напряжение в цепи. Это устройство называют источником тока (генератором). Силы, которые разделяют заряды, называют сторонними силами. Они носят неэлектрическое происхождение и действуют только внутри источника. При разделении зарядов сторонние силы создают разность потенциалов между концами цепи.

В том случае, если электрический заряд перемещается по замкнутой цепи, то работа электростатических сил равна нулю. Значит, суммарная работа сил ($A$), которые действуют на заряд равна работе сторонних сил ($A_{st}$). Физическая величина, которая характеризует источник тока — это ЭДС источника (${\mathcal E}$), она определена как:

\[{\mathcal E}=\frac{A}{q}\left(1\right),\]

где $q$ — положительный заряд. Движение заряда идет по замкнутому контуру. ЭДС — не является силой в буквальном смысле. Единица измерения $\left[{\mathcal E}\right]=В$.

Природа сторонних сил может быть различна, так например, в гальваническом элементе сторонние силы являются результатом электрохимических процессов. В машине постоянного тока такой силой является сила Лоренца.

Основные характеристики тока

Направлением тока условно считают направление движения положительных частиц. Значит, направление тока в металлах имеет противоположное направление по отношению к направлению движения частиц.

Электрический ток характеризуют силой тока. Сила тока ($I$) — скалярная величина, которая равна производной от заряда ($q$) по времени для тока, который течет через поверхность S:

\[I=\frac{dq}{dt}\left(2\right).\]

Ток может быть постоянным и переменным. В том случае, если сила тока и его направление не изменяется во времени, то такой ток называют постоянным и для него выражение для силы тока можно записать в виде:

\[I=\frac{q}{t}\left(3\right),\]

где сила тока определена, как заряд, который проходит через поверхность S в единицу времени.

В системе СИ основной единицей измерения силы тока является Ампер (А).

\[1A=\frac{1Кл}{1с}.\]

Векторной локальной характеристикой тока является его плотность.{-4}\frac{м}{с}$.

Закон сохранения заряда

Рис. 1

Фундаментальным физическим законом является закон сохранения электрического заряда. Если выбрать произвольную замкнутую неподвижную поверхность S (рис.1), которая ограничивает объем V, то количество электричества, которое вытекает за секунду из объема V, определяется как $\oint\limits_S{j_ndS.}$ То же количество электричества можно выразить через заряд: $-\frac{\partial q}{\partial t}$, то есть мы имеем:

\[\frac{\partial q}{\partial t}=-\oint\limits_S{j_ndS\left(6\right),}\]

где $j_n$ — проекция вектора плотности тока на направление нормали к элементу поверхности $dS$, при этом:

\[j_n=jcos\alpha \ \left(7\right),\]

где $\alpha $ — угол между направлением нормали к dS и вектором плотности тока. В уравнении (6) употребляется частная производная для того, чтобы подчеркнуть, что поверхность S неподвижна.

Уравнение (6) — есть закон сохранения заряда в макроскопической электродинамике. В том случае, если ток постоянен во времени, то закон сохранения заряда запишем в виде:

\[\oint\limits_S{j_ndS=0\left(8\right).}\]

Постоянный электрический ток. ОСНОВНЫЕ ПОЛОЖЕНИЯ

Электрический ток — упорядоченное (направленное) движение заряженных частиц Направленное движение свободных зарядов (носителей тока) в проводнике возможно под действием внешнего электрического поля

За направление тока принимается направление движения положительно заряженных частиц.

Электрический ток — упорядоченное (направленное) движение заряженных частиц Направленное движение свободных зарядов (носителей тока) в проводнике возможно под действием внешнего электрического поля

За направление тока принимается направление движения положительно заряженных частиц.

Сила тока в данный момент времени — скалярная физическая величина, равная пределу отношения величины электрического заряда, прошедшего сквозь поперечное сечение проводника, к промежутку времени его прохождения

Единица силы тока (основная единица СИ) — ампер (1 А) 1 А = 1 Кл/с

Постоянный электрический ток — ток, сила которого не изменяется с течением времени

Источник тока — устройство, разделяющее положительные и отрицательные заряды

Сторонние силы — силы неэлектростатического происхождения, вызывающие разделение зарядов в источнике тока

ЭДС— скалярная физическая величина, равная отношению работы сторонних сил по перемещению положительного заряда от отрицательного полюса источника тока к положительному к величине этого заряда:

ЭДС равна напряжению между полюсами разомкнутого источника тока.

Закон Ома для однородного проводника (участка цепи): сила тока в однородном проводнике прямо пропорциональна приложенному напряжению и обратно пропорциональна сопротивлению проводника

Сопротивление проводника прямо пропорционально его удельному сопротивлению и длине и обратно пропорционально площади его поперечного сечения

Единица сопротивления — ом (1 Ом) 1 Ом = 1 В/А

Резистор — проводник с определенным постоянным сопротивлением

Удельное сопротивление — скалярная физическая величина, численно равная сопротивлению однородного цилиндрического проводника единичной длины и единичной площади.

Единица удельного сопротивления — ом-метр (1 Ом • м).

Удельное сопротивление металлического проводника линейно возрастает с температурой:

где ρ0— удельное сопротивление при T0 = 293 К, ΔТ= Т- T0, α — температурный коэффициент сопротивления. Единица температурного коэффициента сопротивления К-1. Удельное сопротивление полупроводника уменьшается при увеличении температуры из-за увеличения числа свободных зарядов, способных переносить электрический ток.

Дырка — вакантное электронное состояние в кристаллической решетке, имеющее избыточный положительный заряд.

Сверхпроводимость — физическое явление, заключающееся в скачкообразном падении до нуля сопротивления вещества.

Критическая температура — температура скачкообразного перехода вещества из нормального состояния в сверхпроводящее.

Изотопический эффект — зависимость критической температуры от массы ионов в кристаллической решетке.

Электрический ток в сверхпроводнике обусловлен согласованным движением пар электронов, связанных между собой взаимодействием с кристаллической решеткой

При последовательном соединении резисторов общее сопротивление цепи равно сумме их сопротивлений При параллельном соединении резисторов проводимость цепи равна сумме их проводимостей Закон Ома для замкнутой цепи: сила тока в замкнутой цепи прямо пропорциональна ЭДС источника и обратно пропорциональна полному сопротивлению цепи:

где R и r — внешнее и внутреннее сопротивления цепи.

Закон Ома для замкнутой цепи с несколькими последовательно соединенными источниками тока:

сила тока в замкнутой цепи с последовательно соединенными источниками тока прямо пропорциональна алгебраической сумме их ЭДС и обратно пропорциональна полному сопротивлению цепи:

Амперметр измеряет силу электрического тока, включается в цепь последовательно

Шунт — проводник, присоединяемый параллельно амперметру для увеличения предела его измерений*

где RA — сопротивление амперметра, n — кратность изменения предела измерений.

Вольтметр измеряет электрическое напряжение. Включается в цепь параллельно

Дополнительное сопротивление — проводник, присоединяемый последовательно с вольтметром для увеличения предела его измерений.

где Rv — сопротивление вольтметра Количество теплоты, выделяющееся в проводнике, равно работе электрического тока.

Закон Джоуля—Ленца: количество теплоты, выделяемое в проводнике с током, равно произведению квадрата силы тока, сопротивления проводника и времени прохождения по нему тока:

Мощность электрического тока — работа, совершаемая в единицу времени электрическим полем при упорядоченном движении заряженных частиц в проводнике

Потребителю передается максимальная мощность, если сопротивление нагрузки равно суммарному сопротивлению источника тока и подводящих проводов

Жидкости, как и твердые тела, могут быть проводниками электрического тока

Электролиты — вещества, растворы и расплавы которых обладают ионной проводимостью.

Электролитическая диссоциация — расщепление молекул электролита на положительные и отрицательные ионы под действием растворителя

Электролиз — выделение на электродах веществ, входящих в состав электролита, при протекании через его раствор (или расплав) электрического тока

Закон Фарадея: масса вещества, выделившегося на электроде, прямо пропорциональна заряду, прошедшему через раствор (расплав) электролита. где k— электрохимический эквивалент вещества.

Единица электрохимического эквивалента — килограмм на кулон (1 кг/Кл).

Объединенный закон Фарадея:

где М — молярная масса, n — валентность химического элемента; постоянная Фарадея F = 9,65- 104Кл/моль.

Electric Current — The Physics Hypertextbook

Обсуждение

определений

текущий

Электрический ток определяется как скорость, с которой заряд протекает через поверхность (например, поперечное сечение провода). Несмотря на то, что оно относится ко многим различным вещам, слово ток часто используется само по себе вместо более длинного, более формального «электрического тока». Прилагательное «электрический» подразумевается контекстом описываемой ситуации.Фраза «ток через тостер», несомненно, относится к потоку электронов через нагревательный элемент, а не к потоку ломтиков хлеба через прорези.

Как и все величины, определяемые как скорость, есть два способа записать определение электрического тока — средний ток для тех, кто заявляет о незнании вычислений…

и мгновенный ток для тех, кто не боится вычислений…

I = q = dq
т дт

Единица измерения тока — ампер [A], названная в честь французского ученого Андре-Мари Ампера (1775–1836).В письменных языках без диакритических букв (а именно в английском) принято писать единицу измерения как ампер , а в неформальном общении сокращать это слово до amp . У меня нет проблем с любым из этих вариантов написания. Только не используйте заглавную букву «А» в начале. Ампер относится к физику, а ампер (или ампер, или ампер) относится к единице.

Поскольку заряд измеряется в кулонах, а время измеряется в секундах, ампер равняется кулону в секунду.



А = С

с

Элементарный заряд определен как ровно…

e = 1,602176634 × 10 −19 C

Число элементарных зарядов в кулонах будет обратной величиной этого числа — повторяющейся десятичной дробью с периодом 778 716 цифр. Я напишу первые 19 цифр, это максимум, что я могу написать (поскольку произвольных долей элементарного заряда не существует).

C ≈ 6,241,509,074,460,762,607 e

А потом напишу еще раз с более разумным количеством цифр, чтобы было легче читать.

C ≈ 6,2415 × 10 18 e

Ток в один ампер — это передача примерно 6,2415 × 10 18 элементарных зарядов в секунду. Для любителей случайностей это примерно десять микромолей.

плотность тока

Когда я визуализирую ток, я вижу, как что-то движется.Я вижу, как они движутся в определенном направлении. Я вижу вектор. Я вижу не то. Ток не является векторной величиной, несмотря на мою хорошо развитую научную интуицию. Ток — это скаляр. И причина в том … потому что это так.

Но подождите, становится еще страннее. Отношение силы тока к площади для данной поверхности называется плотностью тока.

Единица измерения плотности тока — ампер на квадратный метр , не имеющая специального названия.



А = А

м 2 м 2

Несмотря на то, что это отношение двух скалярных величин, плотность тока является вектором.И причина в том, что это так.

Ну… на самом деле, это потому, что плотность тока определяется как произведение плотности заряда и скорости для любого места в космосе…

Дж = ρ v

Два уравнения эквивалентны по величине, как показано ниже.

Дж = ρ в
Дж = q DS = с dq = 1 Я
В дт SA дт А
Дж = Я
А

Есть еще кое-что, что нужно учесть.

I = JA = ρ v A

Читатели, знакомые с механикой жидкостей, могли бы узнать правую часть этого уравнения, если бы оно было написано немного иначе.

I = ρ Av

Это произведение является величиной, которая остается постоянной в уравнении неразрывности массы .

ρ 1 A 1 v 1 = ρ 2 A 2 v 2

Точно такое же выражение применяется к электрическому току с символом ρ, меняющим значение между контекстами.В механике жидкости ρ обозначает массовую плотность, а в электрическом токе — плотность заряда.

микроскопическое описание

Ток — это поток заряженных частиц. Это дискретные сущности, а значит, их можно сосчитать.

Н = Н / В

q = нкВ

В = Ad = Av т

I = q = nqAv т
т т

I = нкАв

Аналогичное выражение можно записать для плотности тока.Вывод начинается в скалярной форме, но в окончательном выражении используются векторы.

Дж = нк в

твердых

проводимость и валентные электроны, проводники и изоляторы

Дрейфовое движение, наложенное на тепловое движение

Увеличить

Мостовой текст.

Тепловая скорость электронов в проводе довольно высока и случайным образом изменяется из-за столкновений атомов. Поскольку изменения хаотичны, средняя скорость равна нулю.

Когда провод помещается в электрическое поле, свободные электроны равномерно ускоряются в промежутках между столкновениями. Эти периоды ускорения поднимают среднюю скорость выше нуля. (Эффект на этой диаграмме сильно преувеличен.)

тепловая скорость электрона в меди при комнатной температуре (классическое приближение)…

v среднеквадратичное значение = √ 3 (1.38 × 10 −23 Дж / К) (300 К)
(9,11 × 10 −31 кг)
v среднеквадратичное значение 100 км / с

Ферми-скорость электрона в меди (квантовая величина)…

v fermi = √ 2 E Ферми
м e
v fermi = √ 2 (7.00 эВ) (1,60 × 10 −19 Дж / эВ)
(9,11 × 10 −31 кг)
v fermi 1500 км / с

Скорость дрейфа электрона на 10 м медного провода, подключенного к автомобильному аккумулятору 12 В при комнатной температуре (среднее время свободного пробега между столкновениями при комнатной температуре τ = 3 × 10 −14 с)…

v дрейф = 1 v = 1 а τ = 1 Ф τ = 1 eE τ
2 2 2 м e 2 м e
v дрейф = (1.60 × 10 −19 C) (12 В) (3 × 10 −14 с)
2 (10 м) (9,11 × 10 −31 кг)
v смещение 3 мм / с

Тепловая скорость на несколько порядков превышает скорость дрейфа в типичной проволоке. Время на прохождение круга — около часа.

жидкости

ионы, электролиты

газы

ионов, плазма

  • 14:02 — Отключение линии электропередачи на юго-западе Огайо
    4. Стюарт — Атланта 345 кВ
    Эта линия является частью пути передачи из юго-западного Огайо в северный Огайо. Он отключился от системы из-за возгорания кисти под частью линии. Горячие газы от пожара могут ионизировать воздух над линией электропередачи, заставляя воздух проводить электричество и закорачивать проводники.
    Источник

исторический

Символ I был выбран французским физиком и математиком Андре-Мари Ампером для обозначения силы тока силы тока.

Увеличить
Pour exprimer en nombre l’intensité d’un courant quelconque, on Concevra qu’on ait choisi un autre courant арбитраж для сравнения терминов…. Désignant donc par i et i раппортов интенсивных деяний двух партнеров, не связанных с интенсивностью, связанной с объединением…. Чтобы выразить интенсивность силы тока в виде числа, предположим, что для сравнения выбран другой произвольный ток…. Используем i и i для отношения интенсивностей двух заданных токов к силе опорного тока, взятого за единицу….
Андре-Мари Ампер, 1826 Андре-Мари Ампер, 1826 г. (платная ссылка)

Термин «интенсивность» теперь не имеет никакого отношения к физике. Ток — это скорость, с которой заряд протекает через поверхность любого размера — например, клеммы батареи или штыри электрической вилки. Интенсивность — это средняя мощность на единицу площади, передаваемая каким-либо явлением излучения — например, звуком оживленного шоссе, светом Солнца или частицами брызг, испускаемыми радиоактивным источником.Ток и интенсивность теперь — разные величины с разными единицами измерения и разным использованием, поэтому (конечно) они используют одинаковые символы.

текущий интенсивность
I = q

А = С

т с
I = п.

Вт

А м 2

Начало таблицы

  • 12000 А ток через магниты LHC в ЦЕРН

Определение электрического тока

Электрический ток — это мера количества электрического заряда, переносимого за единицу времени.Он представляет собой поток электронов через проводящий материал, например металлическую проволоку. Измеряется в амперах.

Единицы и обозначения для электрического тока

Единицей измерения электрического тока в системе СИ является ампер, равный 1 кулону в секунду. Ток — это величина, то есть это одно и то же число независимо от направления потока, без положительного или отрицательного числа. Однако при анализе цепей важно направление тока.

Обычный символ для тока — I , который происходит от французского словосочетания интенсивность куранта , что означает сила тока .Сила тока часто обозначается просто как , ток .

Символ I использовал Андре-Мари Ампер, в честь которого названа единица измерения электрического тока. Он использовал символ I при формулировке закона силы Ампера в 1820 году. Обозначение перешло из Франции в Великобританию, где оно стало стандартом, хотя по крайней мере один журнал не изменился с C на I до 1896 года.

Закон Ома, регулирующий электрический ток

Закон Ома гласит, что ток через проводник между двумя точками прямо пропорционален разности потенциалов в этих двух точках.Вводя константу пропорциональности, сопротивление, приходим к обычному математическому уравнению, описывающему эту взаимосвязь:

I = V / R

В этом соотношении I — это ток через проводник в единицах ампер, В, — это разность потенциалов, измеренная на проводе в единицах вольт, и R — это сопротивление проводника в единицах Ом. . Более конкретно, закон Ома гласит, что R в этом отношении является постоянным и не зависит от тока.Закон Ома используется в электротехнике для решения схем.

Аббревиатуры AC и DC часто используются для обозначения просто переменного и постоянного , когда они изменяют ток или напряжение . Это два основных типа электрического тока.

Постоянный ток

Постоянный ток (DC) — это однонаправленный поток электрического заряда. Электрический заряд течет в постоянном направлении, что отличает его от переменного тока.Термин, ранее использовавшийся для постоянного тока , был гальваническим током.

Постоянный ток вырабатывается такими источниками, как батареи, термопары, солнечные элементы и электрические машины коммутаторного типа динамо-типа. Постоянный ток может течь в проводнике, таком как провод, но также может течь через полупроводники, изоляторы или даже через вакуум, как в электронных или ионных пучках.

Переменный ток

В переменном токе (AC, также AC) движение электрического заряда периодически меняет направление.В постоянном токе электрический заряд идет только в одном направлении.

Переменный ток — это форма подачи электроэнергии на предприятия и жилые дома. Обычная форма волны в цепи питания переменного тока — синусоидальная волна. В некоторых приложениях используются сигналы различной формы, например треугольные или прямоугольные.

Аудио- и радиосигналы, передаваемые по электрическим проводам, также являются примерами переменного тока. Важной целью в этих приложениях является восстановление информации, закодированной (или , модулированной ), в сигнал переменного тока.

Электрический ток

Единица электрического заряда — кулон (сокращенно C). Обычная материя состоит из атомов, которые имеют положительно заряженные ядра и окружающие их отрицательно заряженные электроны. Заряд квантуется как кратное заряду электрона или протона:


Влияние зарядов характеризуется силами между ними (закон Кулона) и создаваемым ими электрическим полем и напряжением.Один кулон заряда — это заряд, который будет проходить через лампочку мощностью 120 ватт (120 вольт переменного тока) за одну секунду. Два заряда одного кулона каждый, разделенный метром, будет отталкивать друг друга с силой около миллиона тонн!

Скорость протекания электрического заряда называется электрическим током и измеряется в амперах.

Представляя одно из фундаментальных свойств материи, возможно, уместно указать, что мы используем упрощенные наброски и конструкции, чтобы представить концепции, и в истории неизбежно гораздо больше.Не имеет значения следует прикрепить к кружкам, представляющим протон и электрон, в чувство подразумевая относительный размер, или даже что они являются твердой сферой объекты, хотя это полезная первая конструкция. Самое важное начальная идея, электрически, это то, что у них есть свойство, называемое «заряд», который одинаковый размер, но противоположные по полярности для протона и электрона. В протон имеет 1836 раз больше массы электрона, но точно такого же размера стоимость только скорее положительный, чем отрицательный.Даже термины «положительный» и «отрицательные» произвольные, но хорошо укоренившиеся исторические ярлыки. Самое важное значение в том, что протон и электрон будут сильно притягивать друг друга. другое — исторический архетип клише «противоположности притягиваются». Два протоны или два электрона сильно отталкиваются друг от друга. Однажды ты имеют установил эти основные представления об электричестве, «как заряды отталкивать и в отличие от обвинений привлекают «, то у вас есть основание для электричество и можно строить оттуда.

Из точной электрической нейтральности объемного вещества, а также из детальных микроскопических экспериментов мы знаем, что протон и электрон имеют одинаковую величину заряда. Все заряды, наблюдаемые в природе, кратны этим фундаментальным зарядам. Хотя стандартная модель протона изображает его состоящим из дробно заряженных частиц, называемых кварками, эти дробные заряды не наблюдаются изолированно — всегда в комбинациях, которые производят +/- заряд электрона.

Изолированный одиночный заряд можно назвать «электрическим монополем». Равные положительный и отрицательный заряды, помещенные близко друг к другу, составляют электрический диполь. Два противоположно направленных диполя, расположенных близко друг к другу, называются электрическим квадруполем. Вы можете продолжить этот процесс для любого количества полюсов, но здесь упоминаются диполи и квадруполи, потому что они находят важное применение в физических явлениях.

Одна из фундаментальных симметрий природы — сохранение электрического заряда.23 и более электронов. Итак, истинное электрическое поле будет прыгать. Но обычно мы говорим не об этом микроскопическом электрическом поле, а о среднем поле. Вы выбираете область, достаточно большую, чтобы в ней было много электронов, но достаточно маленькую, чтобы при небольшом перемещении области среднее (интеграл, деленный на объем) не сильно изменился. Это похоже на плотность населения в городе: вы не хотите, чтобы она увеличивалась на кровати каждого человека или даже в его спальне, вы хотите, чтобы она плавно изменялась из одной части города, чтобы легко видеть, где она относительно плотна.Это означает, что расхождение макроскопически усредненного поля не является буквальной микроскопической плотностью заряда, а опять же плотностью заряда, усредненной по такой же большой области, которая достаточно велика, чтобы иметь много электронов, но достаточно мала, чтобы, если вы немного ее перевели, не получилось. особо не меняю.

Хорошо, мы говорим об этих полях и плотностях заряда. Итак, давайте посмотрим на этот элемент схемы. Если он омический, то это резистор, и обычно нам нужны резисторы, которые удерживают ток внутри него или, по крайней мере, близко к нему.Если есть работа выхода, которая требует некоторой большой энергии, чтобы выбить электрон из металла, и тепловая среда достаточно холодная, так что энергия, если она намного больше, чем тепловое толчки, вероятно, обеспечит, тогда заряды будут течь внутрь, включаться или рядом со стихией, но не сбежать. Таким образом, мы не наблюдаем ускорения заряда в направлении, перпендикулярном проводу, поэтому $ \ vec {E} \ cdot \ vec {n} = 0 $, где $ \ vec {n} $ перпендикулярно границе элемент схемы, или через потенциал $ \ partial V / \ partial n = 0 $.Как насчет частей элемента схемы, не находящихся на границе, частей, подключенных к батарее? Один из них имеет потенциал, один постоянный потенциал, другой, возможно, другой постоянный потенциал. Для простоты и конкретности возьмем за один конец $ (0, r \ sin (\ theta), r \ cos (\ theta)) $ при нулевом потенциале и $ (L, r \ sin (\ theta), r \ cos (\ theta)) $ должен находиться под потенциалом $ V_0 $, поэтому у нас есть короткий прямой круговой провод длиной $ L $ и радиусом $ r $, хотя длина и радиус не важны. Последнее физическое понимание, которое нам нужно, состоит в том, что макроскопическая плотность заряда равна нулю.Если бы оно было ненулевым и макроскопическое электрическое поле было бы ненулевым, то было бы макроскопическое ускорение, а не установившееся состояние. Нестационарные состояния могут быть важными и полезными, но они часто временны, поскольку система приближается к макроскопически устойчивому состоянию, поэтому ищите макроскопически устойчивое состояние, если мы его найдем, то это может быть точное описание этой установки. Итак, теперь мы будем искать макроскопически устойчивое состояние, то есть такое, в котором макроскопическое электрическое поле не имеет расходимости, и у нас есть граничные условия, указанные на всей границе.

Теперь мы можем найти одно решение для электрического потенциала $ V_1 $ внутри резистора, а именно $ V (x, y, z) = xV_0 / L $. Предположим, что существует другое решение $ V_2 $, которое также имеет $ V = 0 $ на одном конце, $ V = V_0 $ на другом и $ \ partial V / \ partial n = 0 $ на краях резистора, а затем рассмотрим два электрических поля $ \ vec {E} _1 = — \ nabla V_1 $ и $ \ vec {E} _2 = — \ nabla V_1 $, затем рассмотрим различия $ \ vec {E} _3 = \ vec {E} _1 — \ vec {E} _2 $ и $ V_3 = V_1-V_2 $, и имейте в виду, что это поля, а не просто числа.2 $ над этим маленьким шариком будет между $ d / 2 $ и $ d $, умноженными на отрицательный объем этого маленького шарика, но интеграл по всей остальной части этого шарика может дать самое большее ноль и, возможно, даже быть отрицательным, поэтому невозможно, чтобы сумма строго отрицательного числа и неположительного числа могла дать вам ноль, поэтому не должно быть случая, чтобы $ \ vec {E} _1 $ и $ \ vec {E} _2 $ были разными, поскольку $ \ vec {E} _3 $ внутри провода равен нулю. И мы знаем, что такое $ \ vec {E} _1 $, это $ (V / L, 0,0) $. Итак, это электрическое поле.Это единообразно. Это единственное макроскопическое поле, которое макроскопически устойчиво.

Должно ли макроскопическое поле быть макроскопически устойчивым? Что ж, у вас есть батарея, и цель батареи — обеспечивать стабильное напряжение, а напряжение — это концепция, зависящая от датчика в постоянно зависимой электродинамике, поэтому, если вы говорите, что у вас есть батарея, и все, что вы хотите сказать о дело в том, что он поддерживает постоянное напряжение, и вы не хотите говорить о том, как он пытается или как он динамически взаимодействует с нестабильным макроскопическим полем, тогда, я думаю, это все, что мы можем сказать.

электромагнетизм — установившийся ток Определение / значение

Я видел 2 (или 3) определения стационарного тока.

Определение 1: $ \ quad \ frac {\ partial} {\ partial t} \ rho = 0 $ или $ \ nabla \ cdot \ mathbf {J} = 0 $

Это означает, как и ожидалось, что ток через произвольную замкнутую поверхность равен $ I = \ oint \ mathbf {J} \, d \ mathbf {S} = 0 $. Если мы рассмотрим бесконечный цилиндр (провод), мы сделаем вывод, что ток через любое перпендикулярное поперечное сечение одинаков.Если $ \ hat {x} $ — направление вдоль цилиндра, $ \ frac {\ partial} {\ partial x} I = 0 $. Мне это кажется, по крайней мере, странным. Поскольку обычно «стационарный» означает постоянство по времени. Википедия (на испанском языке) и статья Милдфорд-Райтц «Основы теории электромагнетизма» поддерживают это определение.

Определение 2: $ \ quad \ frac {\ partial} {\ partial t} \ mathbf {J} = \ vec {0} $

, что означало бы $ \ frac {\ partial} {\ partial t} I = 0 $ для любой поверхности. Однако, если мы рассматриваем замкнутую поверхность и $ I \ neq 0 $, будет накопление ($ I <0 $) или ¿диссипация? ($ I> 0 $) расходов.Это «проблема», которой нельзя избежать из этого определения, не так ли?


Есть третье определение. В параграфе 5.2.1 документа Introduction to Electrodynamics Дэвида Гриффитса автор заявляет

Стационарные заряды ⇒ постоянные электрические поля: электростатика.
Установившиеся токи ⇒ постоянные магнитные поля: магнитостатика.
Под устойчивым током я подразумеваю непрерывный поток, который продолжается вечно, без изменений и без накопления заряда где-либо.(Некоторые люди звонят им «стационарные токи»; на мой слух, это противоречие терминологии.) Формально, Электро / магнитостатика — это режим
∂ρ / ∂t = 0, ∂J / ∂t = 0, (5.32)
везде и всегда.

Отсюда кажется, что автор определяет установившийся ток как оба определения вместе. «без накопления заряда где-либо» (определение 1) и «∂J / ∂t = 0» (определение 2).

Я знаю, что определение 2 является достаточным условием для магнитостатики, определение 1 — для электростатики, и оба необходимы для электромагнетостатики.Мне кажется, что это совершенно разные условия, никак не связанные между собой, с очень разными значениями. Объединение обоих определений могло бы стать решением, но я нигде этого не видел. Я уверен, что мне что-то здесь не хватает.

электричества — Почему ток в цепи не меняется?

Хорошо, это на самом деле довольно легко объяснить без слишком большого количества уравнений и только одной вещи, о которой нужно помнить: заряд не может накапливаться внутри металла .2 $ зависимость электрической силы, с которой электроны действуют друг на друга, пока они снова не будут довольно равномерно распределены по металлу.

Итак, что это означает? Если электроны не могут накапливаться, это означает, что если бы мы измерили скорость прохождения электронов через любую площадь поперечного сечения провода или резистора в цепи (здесь я предполагаю последовательную схему) , он должен быть одинаковым для всех площадей сечения!

Но, по самому определению тока (количество заряда, которое проходит через площадь поперечного сечения металла в секунду) , это означает, что ток должен быть одинаковым везде в цепи (последовательной) ! В противном случае электроны накапливаются.


Теперь я более прямо рассмотрю вашу путаницу: ток через лампочку.

Что происходит у лампочки?

Итак, во-первых, да, вы правы в том, как работает лампа накаливания. Внутри стеклянной колбы находится очень тонкая вольфрамовая нить, и когда через нее протекает ток, она излучает свет. Объясню почему:

Вольфрам обладает двумя свойствами, которые делают его идеальным материалом для создания нити накала лампочки:

  1. Обладает чрезвычайно высокой температурой плавления.
  2. Он довольно проводящий.

Теперь, подобно тому, как вода в одной трубе с двумя секциями разного радиуса течет с большей скоростью через более тонкие секции трубы, если мы сделаем вольфрамовую нить сверхтонкой, электроны должны будут «течь». « (дрейф) быстрее через нить накала, которая тоньше, чем остальная часть цепи, чтобы не было накопления заряда где-либо в цепи. Чем тоньше мы сделаем его, тем с большей скоростью будут проходить электроны, проходя через него!

Sidetrack, но именно поэтому из-за более тонких резисторов падение напряжения больше.Чтобы электроны не накапливались в цепи, они должны проходить быстрее через тонкие части, а для того, чтобы они текли (дрейфовали) быстрее через более тонкие части, должно быть большее электрическое поле, проталкивающее их через тонкие резисторы. . Чем больше электрическое поле, тем больше падение напряжения!

Теперь, поскольку электроны очень быстро движутся через тонкую вольфрамовую нить, они также чаще врезаются в атомы вольфрамовой нити по сравнению с атомами других частей цепи.Это заставляет атомы вольфрама начать очень быстро вибрировать, что нагревает вольфрамовую нить до чрезвычайно высоких температур (поэтому было важно использовать вольфрам или другой металл с высокой температурой плавления) .

Аааи, когда вольфрам нагревается до достаточно высокой температуры, его атомы начинают излучать свет !! (Ааа, я мог бы подробнее рассказать, почему это происходит, если хотите, но я думаю, что это выходит за рамки этого вопроса …)

Надеюсь, что это помогло!

Электрический ток — Университетская физика, том 2

Цели обучения

К концу этого раздела вы сможете:

  • Опишите электрический ток
  • Определите единицу измерения электрического тока
  • Объясните направление тока

До сих пор мы рассматривали в основном статические заряды.Когда заряды действительно двигались, они ускорялись в ответ на электрическое поле, создаваемое разностью напряжений. Заряды теряли потенциальную энергию и приобретали кинетическую энергию, когда они проходили через разность потенциалов, где электрическое поле действовало на заряд.

Хотя заряды не требуют прохождения материала, большая часть этой главы посвящена пониманию движения зарядов через материал. Скорость, с которой заряды проходят мимо места, то есть количество заряда в единицу времени, известна как электрический ток .Когда заряды протекают через среду, ток зависит от приложенного напряжения, материала, через который протекают заряды, и состояния материала. Особый интерес представляет движение зарядов в проводящем проводе. В предыдущих главах заряды ускорялись из-за силы, создаваемой электрическим полем, теряя потенциальную энергию и приобретая кинетическую энергию. В этой главе мы обсуждаем ситуацию силы, создаваемой электрическим полем в проводнике, когда заряды теряют кинетическую энергию в материале, достигая постоянной скорости, известной как « дрейфовая скорость ».Это аналогично тому, как объект, падающий через атмосферу, теряет кинетическую энергию в воздух, достигая постоянной конечной скорости.

Если вы когда-либо проходили курс по оказанию первой помощи или технике безопасности, вы, возможно, слышали, что в случае поражения электрическим током именно ток, а не напряжение, является важным фактором, влияющим на силу удара и количество ударов. повреждение человеческого тела. Ток измеряется в единицах, называемых амперами; Возможно, вы заметили, что автоматические выключатели в вашем доме и предохранители в машине имеют номинал в амперах (или амперах).Но что такое ампер и что он измеряет?

Определение тока и ампера

Электрический ток определяется как скорость, с которой протекает заряд. Когда присутствует большой ток, например, используемый для работы холодильника, большое количество заряда перемещается по проводу за небольшой промежуток времени. Если ток небольшой, например, используемый для работы портативного калькулятора, небольшое количество заряда перемещается по цепи в течение длительного периода времени.

Большинство электроприборов рассчитаны в амперах (или амперах), необходимых для правильной работы, равно как и предохранители и автоматические выключатели.

Скорость потока заряда текущая. Ампер — это поток одного кулона заряда через область за одну секунду. Ток в один ампер будет результатом протекания электронов через область A каждую секунду.

Расчет среднего тока Основное назначение аккумуляторной батареи в легковом или грузовом автомобиле — запускать электрический стартер, который запускает двигатель. Для запуска двигателя требуется большой ток, подаваемый аккумулятором. После запуска двигателя устройство, называемое генератором переменного тока, берет на себя подачу электроэнергии, необходимой для работы транспортного средства и для зарядки аккумулятора.

(a) Какой средний ток возникает, когда аккумулятор грузового автомобиля приводит в движение 720 C заряда за 4,00 с при запуске двигателя? (b) Сколько времени требуется 1,00 C для зарядки аккумулятора?

Стратегия

Мы можем использовать определение среднего тока в уравнении, чтобы найти средний ток в части (а), поскольку даны заряд и время. Что касается части (b), когда мы знаем средний ток, мы можем определить его, чтобы найти время, необходимое для того, чтобы заряд 1,00 C прошел от батареи.

Решение а. Ввод данных значений заряда и времени в определение тока дает

г. Решение зависимости времени и ввод известных значений заряда и тока дает

Значение а. Это большое значение тока иллюстрирует тот факт, что большой заряд перемещается за небольшой промежуток времени. Токи в этих «стартерных двигателях» довольно велики, чтобы преодолеть инерцию двигателя. б. Сильный ток требует короткого времени для подачи большого количества заряда.Этот большой ток необходим для подачи большого количества энергии, необходимой для запуска двигателя.

Проверьте свое понимание В портативных калькуляторах часто используются небольшие солнечные батареи для обеспечения энергии, необходимой для выполнения расчетов, необходимых для выполнения следующего экзамена по физике. Ток, необходимый для работы вашего калькулятора, может составлять всего 0,30 мА. Сколько времени потребуется, чтобы заряд 1,00 C потек из солнечных элементов? Можно ли использовать солнечные элементы вместо батарей для запуска традиционных двигателей внутреннего сгорания, которые в настоящее время используются в большинстве легковых и грузовых автомобилей?

Время для 1.00 C заряда для протекания будет, чуть меньше часа. Это сильно отличается от 5,55 мс для аккумулятора грузовика. Калькулятор требует очень мало энергии для работы, в отличие от стартера грузовика. Есть несколько причин, по которым в автомобилях используются батареи, а не солнечные элементы. Помимо очевидного факта, что источник света для запуска солнечных элементов в автомобиле или грузовике не всегда доступен, большое количество тока, необходимого для запуска двигателя, не может быть легко обеспечено современными солнечными элементами.Солнечные элементы могут быть использованы для зарядки батарей. Зарядка аккумулятора требует небольшого количества энергии по сравнению с энергией, необходимой для работы двигателя и других аксессуаров, таких как обогреватель и кондиционер. Современные автомобили на солнечных батареях питаются от солнечных батарей, которые могут приводить в действие электродвигатель, а не двигатель внутреннего сгорания.

Проверьте свое понимание Автоматические выключатели в домашних условиях имеют номинал в амперах, обычно в диапазоне от 10 до 30 ампер, и используются для защиты жителей от повреждений, а их электроприборы — от повреждений из-за больших токов.Один автоматический выключатель на 15 А можно использовать для защиты нескольких розеток в гостиной, а один автоматический выключатель на 20 А можно использовать для защиты холодильника на кухне. Что вы можете сделать из этого о токе, используемом различными приборами?

Суммарный ток, необходимый всем приборам в гостиной (несколько ламп, телевизор и ваш ноутбук), потребляет меньше тока и потребляет меньше энергии, чем холодильник.

Ток в цепи

В предыдущих параграфах мы определили ток как заряд, который проходит через площадь поперечного сечения в единицу времени.Для прохождения заряда через прибор, такой как фара, показанная на (Рисунок), должен быть полный путь (или цепь) от положительной клеммы к отрицательной. Рассмотрим простую схему автомобильного аккумулятора, выключателя, лампы фары и проводов, обеспечивающих ток между компонентами. Для того, чтобы лампа загорелась, должен быть полный путь прохождения тока. Другими словами, заряд должен иметь возможность покинуть положительную клемму батареи, пройти через компонент и вернуться к отрицательной клемме батареи.Переключатель предназначен для управления цепью. На части (а) рисунка показана простая схема автомобильного аккумулятора, выключателя, токопроводящей дорожки и лампы фары. Также показана схема [часть (b)]. Схема — это графическое представление схемы, которое очень полезно для визуализации основных характеристик схемы. На схемах используются стандартные символы для обозначения компонентов в цепях и сплошные линии для обозначения проводов, соединяющих компоненты. Батарея показана в виде серии длинных и коротких линий, представляющих историческую гальваническую батарею.Лампа изображена в виде круга с петлей внутри, что представляет собой нить накаливания. Переключатель показан в виде двух точек с проводящей полосой для соединения этих двух точек, а провода, соединяющие компоненты, показаны сплошными линиями. Схема в части (c) показывает направление тока, когда переключатель замкнут.

(а) Простая электрическая схема фары (лампы), аккумулятора и переключателя. Когда переключатель замкнут, непрерывный путь для прохождения тока обеспечивается проводящими проводами, соединяющими нагрузку с выводами батареи.(b) На этой схеме батарея представлена ​​параллельными линиями, которые напоминают пластины в оригинальной конструкции батареи. Более длинные линии указывают на положительный вывод. Проводящие провода показаны сплошными линиями. Переключатель показан в разомкнутом положении в виде двух клемм с линией, представляющей токопроводящую шину, которая может контактировать между двумя клеммами. Лампа представлена ​​кружком, охватывающим нить накаливания, как если бы это была лампа накаливания. (c) Когда переключатель замкнут, цепь замыкается, и ток течет от положительной клеммы к отрицательной клемме батареи.

Когда переключатель замкнут на (Рисунок) (c), существует полный путь для прохождения зарядов от положительной клеммы батареи через переключатель, затем через фару и обратно к отрицательной клемме батареи. Обратите внимание, что направление тока — от положительного к отрицательному. Направление обычного тока всегда представлено в направлении протекания положительного заряда от положительного вывода к отрицательному.

Обычный ток течет от положительной клеммы к отрицательной, но в зависимости от реальной ситуации положительные заряды, отрицательные заряды или и то, и другое могут перемещаться.В металлических проводах, например, ток переносится электронами, то есть движутся отрицательные заряды. В ионных растворах, таких как соленая вода, движутся как положительные, так и отрицательные заряды. То же самое и с нервными клетками. Генератор Ван де Граафа, используемый для ядерных исследований, может производить ток чисто положительных зарядов, таких как протоны. В ускорителе Тэватрон в Фермилабе, до его закрытия в 2011 году, сталкивались пучки протонов и антипротонов, движущихся в противоположных направлениях. Протоны положительны и, следовательно, их ток в том же направлении, в котором они движутся.Антипротоны заряжены отрицательно, и, следовательно, их ток идет в направлении, противоположном направлению движения реальных частиц.

Более пристальный взгляд на ток, протекающий по проводу, показан на (Рисунок). На рисунке показано движение заряженных частиц, составляющих ток. Тот факт, что обычный ток считается направленным в направлении протекания положительного заряда, можно проследить до американского ученого и государственного деятеля Бенджамина Франклина 1700-х годов. Не зная о частицах, составляющих атом (а именно о протоне, электроне и нейтроне), Франклин полагал, что электрический ток течет от материала, в котором больше «электрической жидкости», и к материалу, в котором этого «меньше». электрическая жидкость.Он ввел термин положительный для материала, в котором больше этой электрической жидкости, и отрицательный для материала, в котором отсутствует электрическая жидкость. Он предположил, что ток будет течь от материала с большим количеством электрической жидкости — положительного материала — к отрицательному материалу, в котором меньше электрической жидкости. Франклин назвал это направление тока положительным током. Это было довольно продвинутое мышление для человека, который ничего не знал об атоме.

Ток I — это скорость, с которой заряд движется через область A , такую ​​как поперечное сечение провода.Обычный ток определяется движением в направлении электрического поля. (а) Положительные заряды движутся в направлении электрического поля, которое совпадает с направлением обычного тока. (б) Отрицательные заряды движутся в направлении, противоположном электрическому полю. Обычный ток идет в направлении, противоположном движению отрицательного заряда. Поток электронов иногда называют электронным потоком.

Теперь мы знаем, что материал является положительным, если в нем больше протонов, чем электронов, и отрицательным, если в нем больше электронов, чем протонов.В проводящем металле ток в основном возникает из-за того, что электроны текут от отрицательного материала к положительному, но по историческим причинам мы рассматриваем положительный ток, и ток, как показано, течет от положительного вывода батареи к положительному. отрицательный терминал.

Важно понимать, что электрическое поле присутствует в проводниках и отвечает за производство тока ((рисунок)). В предыдущих главах мы рассматривали случай статического электричества, когда заряды в проводнике быстро перераспределяются по поверхности проводника, чтобы нейтрализовать внешнее электрическое поле и восстановить равновесие.В случае электрической цепи заряды никогда не достигают равновесия с помощью внешнего источника электрического потенциала, такого как батарея. Энергия, необходимая для перемещения заряда, обеспечивается электрическим потенциалом от батареи.

Хотя электрическое поле отвечает за движение зарядов в проводнике, работа, совершаемая над зарядами электрическим полем, не увеличивает кинетическую энергию зарядов. Мы покажем, что электрическое поле отвечает за поддержание движения электрических зарядов с «дрейфовой скоростью».”

Сводка

  • Средний электрический ток — это скорость протекания заряда, определяемая выражением, где — количество заряда, проходящего через область во времени.
  • Мгновенный электрический ток или просто ток I — это скорость, с которой протекает заряд. Принимая предел, когда изменение во времени приближается к нулю, мы имеем, где — производная заряда по времени.
  • Направление обычного тока принимается за направление, в котором движется положительный заряд.В простой цепи постоянного тока (DC) это будет от положительной клеммы батареи к отрицательной.
  • Единицей измерения тока в системе СИ является ампер или просто ампер (А), где.
  • Ток состоит из потока свободных зарядов, таких как электроны, протоны и ионы.

Концептуальные вопросы

Может ли провод пропускать ток и оставаться нейтральным, то есть иметь нулевой общий заряд? Объяснять.

Если по проводу протекает ток, заряды входят в провод с положительной клеммы источника напряжения и уходят с отрицательной клеммы, поэтому общий заряд остается нулевым, пока через него протекает ток.

Автомобильные аккумуляторы указаны в ампер-часах. Какой физической величине соответствуют ампер-часы (напряжение, ток, заряд, энергия, мощность,…)?

При работе с мощными электрическими цепями рекомендуется по возможности работать «одной рукой» или «держать одну руку в кармане». Почему это разумное предложение?

Использование одной руки снижает вероятность «замыкания цепи» и протекания тока через ваше тело, особенно тока, протекающего через ваше сердце.

Глоссарий

ампер (ампер)
единица СИ для тока;
цепь
полный путь, по которому электрический ток проходит по
условного тока
ток, который течет по цепи от положительной клеммы батареи через цепь к отрицательной клемме батареи
электрический ток
скорость, по которой течет заряд,
схема
графическое представление схемы с использованием стандартных символов для компонентов и сплошных линий для провода, соединяющего компоненты
.

alexxlab

Добавить комментарий

Ваш адрес email не будет опубликован. Обязательные поля помечены *